How to calculate the surface area of parametric surface?











up vote
3
down vote

favorite












Suppose you have the surface $z=3xy$ and you want to find the area that lies within the cylinder $x^2+y^2leq 1$.



My homework is forcing me to use the parameterization



$$textbf{r}_1(s,t)= <scos(t), ssin(t), 3s^2sin(t)cos(t)>$$



I am having a difficult time visualizing this parameterization, and I do not have any graphing software to graph the surface, but I want to make sure I understand this concept.



This is quite obvious, but I want to be sure; using the above parameterization, I am not parameterizing the entire surface, right? If I wanted to, I assume the parameterization would be $$textbf{r}_2(s,t) = <s,t,3st>$$



Instead, is $textbf{r}_1$ just the parameterization adjusted for the region - the region being the cylinder $x^2+y^2leq 1$? That is, are we just making a revolution around $z=3xy$?



Any insight would be helpful.










share|cite|improve this question


























    up vote
    3
    down vote

    favorite












    Suppose you have the surface $z=3xy$ and you want to find the area that lies within the cylinder $x^2+y^2leq 1$.



    My homework is forcing me to use the parameterization



    $$textbf{r}_1(s,t)= <scos(t), ssin(t), 3s^2sin(t)cos(t)>$$



    I am having a difficult time visualizing this parameterization, and I do not have any graphing software to graph the surface, but I want to make sure I understand this concept.



    This is quite obvious, but I want to be sure; using the above parameterization, I am not parameterizing the entire surface, right? If I wanted to, I assume the parameterization would be $$textbf{r}_2(s,t) = <s,t,3st>$$



    Instead, is $textbf{r}_1$ just the parameterization adjusted for the region - the region being the cylinder $x^2+y^2leq 1$? That is, are we just making a revolution around $z=3xy$?



    Any insight would be helpful.










    share|cite|improve this question
























      up vote
      3
      down vote

      favorite









      up vote
      3
      down vote

      favorite











      Suppose you have the surface $z=3xy$ and you want to find the area that lies within the cylinder $x^2+y^2leq 1$.



      My homework is forcing me to use the parameterization



      $$textbf{r}_1(s,t)= <scos(t), ssin(t), 3s^2sin(t)cos(t)>$$



      I am having a difficult time visualizing this parameterization, and I do not have any graphing software to graph the surface, but I want to make sure I understand this concept.



      This is quite obvious, but I want to be sure; using the above parameterization, I am not parameterizing the entire surface, right? If I wanted to, I assume the parameterization would be $$textbf{r}_2(s,t) = <s,t,3st>$$



      Instead, is $textbf{r}_1$ just the parameterization adjusted for the region - the region being the cylinder $x^2+y^2leq 1$? That is, are we just making a revolution around $z=3xy$?



      Any insight would be helpful.










      share|cite|improve this question













      Suppose you have the surface $z=3xy$ and you want to find the area that lies within the cylinder $x^2+y^2leq 1$.



      My homework is forcing me to use the parameterization



      $$textbf{r}_1(s,t)= <scos(t), ssin(t), 3s^2sin(t)cos(t)>$$



      I am having a difficult time visualizing this parameterization, and I do not have any graphing software to graph the surface, but I want to make sure I understand this concept.



      This is quite obvious, but I want to be sure; using the above parameterization, I am not parameterizing the entire surface, right? If I wanted to, I assume the parameterization would be $$textbf{r}_2(s,t) = <s,t,3st>$$



      Instead, is $textbf{r}_1$ just the parameterization adjusted for the region - the region being the cylinder $x^2+y^2leq 1$? That is, are we just making a revolution around $z=3xy$?



      Any insight would be helpful.







      multivariable-calculus vectors surfaces parametrization






      share|cite|improve this question













      share|cite|improve this question











      share|cite|improve this question




      share|cite|improve this question










      asked 19 hours ago









      Art

      3037




      3037






















          2 Answers
          2






          active

          oldest

          votes

















          up vote
          1
          down vote



          accepted










          You can see this exercise as a surface integral or you can also see it as a double integral of a function $f(x,y)$ over the unit disc.





          $mathbf r_1$ is the cylindrical parameterization, which, generally, is
          $$begin{cases}x=scos t\y=ssin t\z=f(x,y)=f(scos t,ssin t)end{cases}$$



          In your case $z=f(x,y)=3xy$. In $mathbf r_1$, the cylinder becomes $sle1$ and you have no limitations on $t$. You have to evaluate the integral
          $$begin{align}int_0^1stimes 3s^2mathrm dsint_0^{2pi}sin(t)cos(t)mathrm dt&=0end{align}$$



          I got to this integral by the following way:



          $$iint f(x,y)mathop{mathrm dx}mathop{mathrm dy}=iint 3xymathop{mathrm dx}mathop{mathrm dy}$$
          Using polar coordinates $(x=scos t, y=ssin t)$ and adding the Jacobian:
          $$iint3s^3sin tcos tmathop{mathrm ds}mathop{mathrm dt}$$
          Because we are in the unit disc, $0<sle1, quad0<t<2pi$
          $$int_0^13s^3left(int_0^{2pi}sin tcos tmathop{mathrm dt}right)mathop{mathrm ds}$$






          share|cite|improve this answer























          • The three upvotes and acceptance notwithstanding this answer is seriously wrong. Why should the area in question be $=0>$? We have a real "floppy disc" here!
            – Christian Blatter
            17 hours ago










          • I think the integrand is right, the limits of integration could be wrong. I tried to evaluate the integral in cartesian coordinates and I got $0$ again: $oint_{x^2+y^2le1}3xymathop{mathrm dxmathrm dy}=3int_{-1}^1xmathop{mathrm dx} int_{-sqrt{1-x^2}}^{sqrt{1-x^2}}ymathop{mathrm dy}=0$. I honestly do not see where the mistake is
            – Lorenzo B.
            17 hours ago












          • I understand your answer (+1). Why mine is wrong?
            – Lorenzo B.
            14 hours ago










          • What made you arrive at the integrand $3s^3cos tsin t>$? – That's my last word on this matter.
            – Christian Blatter
            14 hours ago




















          up vote
          4
          down vote













          Since in the end you have to integrate over the unit disc in the $(x,y)$-plane your source proposes to use polar coordinates instead of $x$ and $y$ as parameters. Then $x=scos t$, $y=ssin t$. In this way the idea $z=3xy$ $>(x^2+y^2leq1)$ translates into
          $${bf r}(s,t)=(scos t,ssin t,3s^2cos tsin t)qquad(0leq sleq 1, 0leq tleq2pi) .$$
          In order to find the area of this floppy disc $F$ we have to compute
          $${bf r}_s=(cos t,sin t, 6scos tsin t),quad {bf r}_t=bigl(-ssin t,scos t ,3s^2cos(2t)bigr)$$
          and then
          $${bf r}_stimes{bf r}_t=(ldots,ldots,ldots) .$$
          The area is then finally given as
          $${rm area}(F)=int_0^1int_0^{2pi}bigl|{bf r}_stimes{bf r}_tbigr|>dt>ds .$$
          The resulting integral will be simpler than dreaded.






          share|cite|improve this answer























            Your Answer





            StackExchange.ifUsing("editor", function () {
            return StackExchange.using("mathjaxEditing", function () {
            StackExchange.MarkdownEditor.creationCallbacks.add(function (editor, postfix) {
            StackExchange.mathjaxEditing.prepareWmdForMathJax(editor, postfix, [["$", "$"], ["\\(","\\)"]]);
            });
            });
            }, "mathjax-editing");

            StackExchange.ready(function() {
            var channelOptions = {
            tags: "".split(" "),
            id: "69"
            };
            initTagRenderer("".split(" "), "".split(" "), channelOptions);

            StackExchange.using("externalEditor", function() {
            // Have to fire editor after snippets, if snippets enabled
            if (StackExchange.settings.snippets.snippetsEnabled) {
            StackExchange.using("snippets", function() {
            createEditor();
            });
            }
            else {
            createEditor();
            }
            });

            function createEditor() {
            StackExchange.prepareEditor({
            heartbeatType: 'answer',
            convertImagesToLinks: true,
            noModals: true,
            showLowRepImageUploadWarning: true,
            reputationToPostImages: 10,
            bindNavPrevention: true,
            postfix: "",
            imageUploader: {
            brandingHtml: "Powered by u003ca class="icon-imgur-white" href="https://imgur.com/"u003eu003c/au003e",
            contentPolicyHtml: "User contributions licensed under u003ca href="https://creativecommons.org/licenses/by-sa/3.0/"u003ecc by-sa 3.0 with attribution requiredu003c/au003e u003ca href="https://stackoverflow.com/legal/content-policy"u003e(content policy)u003c/au003e",
            allowUrls: true
            },
            noCode: true, onDemand: true,
            discardSelector: ".discard-answer"
            ,immediatelyShowMarkdownHelp:true
            });


            }
            });














             

            draft saved


            draft discarded


















            StackExchange.ready(
            function () {
            StackExchange.openid.initPostLogin('.new-post-login', 'https%3a%2f%2fmath.stackexchange.com%2fquestions%2f3004766%2fhow-to-calculate-the-surface-area-of-parametric-surface%23new-answer', 'question_page');
            }
            );

            Post as a guest















            Required, but never shown

























            2 Answers
            2






            active

            oldest

            votes








            2 Answers
            2






            active

            oldest

            votes









            active

            oldest

            votes






            active

            oldest

            votes








            up vote
            1
            down vote



            accepted










            You can see this exercise as a surface integral or you can also see it as a double integral of a function $f(x,y)$ over the unit disc.





            $mathbf r_1$ is the cylindrical parameterization, which, generally, is
            $$begin{cases}x=scos t\y=ssin t\z=f(x,y)=f(scos t,ssin t)end{cases}$$



            In your case $z=f(x,y)=3xy$. In $mathbf r_1$, the cylinder becomes $sle1$ and you have no limitations on $t$. You have to evaluate the integral
            $$begin{align}int_0^1stimes 3s^2mathrm dsint_0^{2pi}sin(t)cos(t)mathrm dt&=0end{align}$$



            I got to this integral by the following way:



            $$iint f(x,y)mathop{mathrm dx}mathop{mathrm dy}=iint 3xymathop{mathrm dx}mathop{mathrm dy}$$
            Using polar coordinates $(x=scos t, y=ssin t)$ and adding the Jacobian:
            $$iint3s^3sin tcos tmathop{mathrm ds}mathop{mathrm dt}$$
            Because we are in the unit disc, $0<sle1, quad0<t<2pi$
            $$int_0^13s^3left(int_0^{2pi}sin tcos tmathop{mathrm dt}right)mathop{mathrm ds}$$






            share|cite|improve this answer























            • The three upvotes and acceptance notwithstanding this answer is seriously wrong. Why should the area in question be $=0>$? We have a real "floppy disc" here!
              – Christian Blatter
              17 hours ago










            • I think the integrand is right, the limits of integration could be wrong. I tried to evaluate the integral in cartesian coordinates and I got $0$ again: $oint_{x^2+y^2le1}3xymathop{mathrm dxmathrm dy}=3int_{-1}^1xmathop{mathrm dx} int_{-sqrt{1-x^2}}^{sqrt{1-x^2}}ymathop{mathrm dy}=0$. I honestly do not see where the mistake is
              – Lorenzo B.
              17 hours ago












            • I understand your answer (+1). Why mine is wrong?
              – Lorenzo B.
              14 hours ago










            • What made you arrive at the integrand $3s^3cos tsin t>$? – That's my last word on this matter.
              – Christian Blatter
              14 hours ago

















            up vote
            1
            down vote



            accepted










            You can see this exercise as a surface integral or you can also see it as a double integral of a function $f(x,y)$ over the unit disc.





            $mathbf r_1$ is the cylindrical parameterization, which, generally, is
            $$begin{cases}x=scos t\y=ssin t\z=f(x,y)=f(scos t,ssin t)end{cases}$$



            In your case $z=f(x,y)=3xy$. In $mathbf r_1$, the cylinder becomes $sle1$ and you have no limitations on $t$. You have to evaluate the integral
            $$begin{align}int_0^1stimes 3s^2mathrm dsint_0^{2pi}sin(t)cos(t)mathrm dt&=0end{align}$$



            I got to this integral by the following way:



            $$iint f(x,y)mathop{mathrm dx}mathop{mathrm dy}=iint 3xymathop{mathrm dx}mathop{mathrm dy}$$
            Using polar coordinates $(x=scos t, y=ssin t)$ and adding the Jacobian:
            $$iint3s^3sin tcos tmathop{mathrm ds}mathop{mathrm dt}$$
            Because we are in the unit disc, $0<sle1, quad0<t<2pi$
            $$int_0^13s^3left(int_0^{2pi}sin tcos tmathop{mathrm dt}right)mathop{mathrm ds}$$






            share|cite|improve this answer























            • The three upvotes and acceptance notwithstanding this answer is seriously wrong. Why should the area in question be $=0>$? We have a real "floppy disc" here!
              – Christian Blatter
              17 hours ago










            • I think the integrand is right, the limits of integration could be wrong. I tried to evaluate the integral in cartesian coordinates and I got $0$ again: $oint_{x^2+y^2le1}3xymathop{mathrm dxmathrm dy}=3int_{-1}^1xmathop{mathrm dx} int_{-sqrt{1-x^2}}^{sqrt{1-x^2}}ymathop{mathrm dy}=0$. I honestly do not see where the mistake is
              – Lorenzo B.
              17 hours ago












            • I understand your answer (+1). Why mine is wrong?
              – Lorenzo B.
              14 hours ago










            • What made you arrive at the integrand $3s^3cos tsin t>$? – That's my last word on this matter.
              – Christian Blatter
              14 hours ago















            up vote
            1
            down vote



            accepted







            up vote
            1
            down vote



            accepted






            You can see this exercise as a surface integral or you can also see it as a double integral of a function $f(x,y)$ over the unit disc.





            $mathbf r_1$ is the cylindrical parameterization, which, generally, is
            $$begin{cases}x=scos t\y=ssin t\z=f(x,y)=f(scos t,ssin t)end{cases}$$



            In your case $z=f(x,y)=3xy$. In $mathbf r_1$, the cylinder becomes $sle1$ and you have no limitations on $t$. You have to evaluate the integral
            $$begin{align}int_0^1stimes 3s^2mathrm dsint_0^{2pi}sin(t)cos(t)mathrm dt&=0end{align}$$



            I got to this integral by the following way:



            $$iint f(x,y)mathop{mathrm dx}mathop{mathrm dy}=iint 3xymathop{mathrm dx}mathop{mathrm dy}$$
            Using polar coordinates $(x=scos t, y=ssin t)$ and adding the Jacobian:
            $$iint3s^3sin tcos tmathop{mathrm ds}mathop{mathrm dt}$$
            Because we are in the unit disc, $0<sle1, quad0<t<2pi$
            $$int_0^13s^3left(int_0^{2pi}sin tcos tmathop{mathrm dt}right)mathop{mathrm ds}$$






            share|cite|improve this answer














            You can see this exercise as a surface integral or you can also see it as a double integral of a function $f(x,y)$ over the unit disc.





            $mathbf r_1$ is the cylindrical parameterization, which, generally, is
            $$begin{cases}x=scos t\y=ssin t\z=f(x,y)=f(scos t,ssin t)end{cases}$$



            In your case $z=f(x,y)=3xy$. In $mathbf r_1$, the cylinder becomes $sle1$ and you have no limitations on $t$. You have to evaluate the integral
            $$begin{align}int_0^1stimes 3s^2mathrm dsint_0^{2pi}sin(t)cos(t)mathrm dt&=0end{align}$$



            I got to this integral by the following way:



            $$iint f(x,y)mathop{mathrm dx}mathop{mathrm dy}=iint 3xymathop{mathrm dx}mathop{mathrm dy}$$
            Using polar coordinates $(x=scos t, y=ssin t)$ and adding the Jacobian:
            $$iint3s^3sin tcos tmathop{mathrm ds}mathop{mathrm dt}$$
            Because we are in the unit disc, $0<sle1, quad0<t<2pi$
            $$int_0^13s^3left(int_0^{2pi}sin tcos tmathop{mathrm dt}right)mathop{mathrm ds}$$







            share|cite|improve this answer














            share|cite|improve this answer



            share|cite|improve this answer








            edited 14 hours ago

























            answered 18 hours ago









            Lorenzo B.

            1,6222419




            1,6222419












            • The three upvotes and acceptance notwithstanding this answer is seriously wrong. Why should the area in question be $=0>$? We have a real "floppy disc" here!
              – Christian Blatter
              17 hours ago










            • I think the integrand is right, the limits of integration could be wrong. I tried to evaluate the integral in cartesian coordinates and I got $0$ again: $oint_{x^2+y^2le1}3xymathop{mathrm dxmathrm dy}=3int_{-1}^1xmathop{mathrm dx} int_{-sqrt{1-x^2}}^{sqrt{1-x^2}}ymathop{mathrm dy}=0$. I honestly do not see where the mistake is
              – Lorenzo B.
              17 hours ago












            • I understand your answer (+1). Why mine is wrong?
              – Lorenzo B.
              14 hours ago










            • What made you arrive at the integrand $3s^3cos tsin t>$? – That's my last word on this matter.
              – Christian Blatter
              14 hours ago




















            • The three upvotes and acceptance notwithstanding this answer is seriously wrong. Why should the area in question be $=0>$? We have a real "floppy disc" here!
              – Christian Blatter
              17 hours ago










            • I think the integrand is right, the limits of integration could be wrong. I tried to evaluate the integral in cartesian coordinates and I got $0$ again: $oint_{x^2+y^2le1}3xymathop{mathrm dxmathrm dy}=3int_{-1}^1xmathop{mathrm dx} int_{-sqrt{1-x^2}}^{sqrt{1-x^2}}ymathop{mathrm dy}=0$. I honestly do not see where the mistake is
              – Lorenzo B.
              17 hours ago












            • I understand your answer (+1). Why mine is wrong?
              – Lorenzo B.
              14 hours ago










            • What made you arrive at the integrand $3s^3cos tsin t>$? – That's my last word on this matter.
              – Christian Blatter
              14 hours ago


















            The three upvotes and acceptance notwithstanding this answer is seriously wrong. Why should the area in question be $=0>$? We have a real "floppy disc" here!
            – Christian Blatter
            17 hours ago




            The three upvotes and acceptance notwithstanding this answer is seriously wrong. Why should the area in question be $=0>$? We have a real "floppy disc" here!
            – Christian Blatter
            17 hours ago












            I think the integrand is right, the limits of integration could be wrong. I tried to evaluate the integral in cartesian coordinates and I got $0$ again: $oint_{x^2+y^2le1}3xymathop{mathrm dxmathrm dy}=3int_{-1}^1xmathop{mathrm dx} int_{-sqrt{1-x^2}}^{sqrt{1-x^2}}ymathop{mathrm dy}=0$. I honestly do not see where the mistake is
            – Lorenzo B.
            17 hours ago






            I think the integrand is right, the limits of integration could be wrong. I tried to evaluate the integral in cartesian coordinates and I got $0$ again: $oint_{x^2+y^2le1}3xymathop{mathrm dxmathrm dy}=3int_{-1}^1xmathop{mathrm dx} int_{-sqrt{1-x^2}}^{sqrt{1-x^2}}ymathop{mathrm dy}=0$. I honestly do not see where the mistake is
            – Lorenzo B.
            17 hours ago














            I understand your answer (+1). Why mine is wrong?
            – Lorenzo B.
            14 hours ago




            I understand your answer (+1). Why mine is wrong?
            – Lorenzo B.
            14 hours ago












            What made you arrive at the integrand $3s^3cos tsin t>$? – That's my last word on this matter.
            – Christian Blatter
            14 hours ago






            What made you arrive at the integrand $3s^3cos tsin t>$? – That's my last word on this matter.
            – Christian Blatter
            14 hours ago












            up vote
            4
            down vote













            Since in the end you have to integrate over the unit disc in the $(x,y)$-plane your source proposes to use polar coordinates instead of $x$ and $y$ as parameters. Then $x=scos t$, $y=ssin t$. In this way the idea $z=3xy$ $>(x^2+y^2leq1)$ translates into
            $${bf r}(s,t)=(scos t,ssin t,3s^2cos tsin t)qquad(0leq sleq 1, 0leq tleq2pi) .$$
            In order to find the area of this floppy disc $F$ we have to compute
            $${bf r}_s=(cos t,sin t, 6scos tsin t),quad {bf r}_t=bigl(-ssin t,scos t ,3s^2cos(2t)bigr)$$
            and then
            $${bf r}_stimes{bf r}_t=(ldots,ldots,ldots) .$$
            The area is then finally given as
            $${rm area}(F)=int_0^1int_0^{2pi}bigl|{bf r}_stimes{bf r}_tbigr|>dt>ds .$$
            The resulting integral will be simpler than dreaded.






            share|cite|improve this answer



























              up vote
              4
              down vote













              Since in the end you have to integrate over the unit disc in the $(x,y)$-plane your source proposes to use polar coordinates instead of $x$ and $y$ as parameters. Then $x=scos t$, $y=ssin t$. In this way the idea $z=3xy$ $>(x^2+y^2leq1)$ translates into
              $${bf r}(s,t)=(scos t,ssin t,3s^2cos tsin t)qquad(0leq sleq 1, 0leq tleq2pi) .$$
              In order to find the area of this floppy disc $F$ we have to compute
              $${bf r}_s=(cos t,sin t, 6scos tsin t),quad {bf r}_t=bigl(-ssin t,scos t ,3s^2cos(2t)bigr)$$
              and then
              $${bf r}_stimes{bf r}_t=(ldots,ldots,ldots) .$$
              The area is then finally given as
              $${rm area}(F)=int_0^1int_0^{2pi}bigl|{bf r}_stimes{bf r}_tbigr|>dt>ds .$$
              The resulting integral will be simpler than dreaded.






              share|cite|improve this answer

























                up vote
                4
                down vote










                up vote
                4
                down vote









                Since in the end you have to integrate over the unit disc in the $(x,y)$-plane your source proposes to use polar coordinates instead of $x$ and $y$ as parameters. Then $x=scos t$, $y=ssin t$. In this way the idea $z=3xy$ $>(x^2+y^2leq1)$ translates into
                $${bf r}(s,t)=(scos t,ssin t,3s^2cos tsin t)qquad(0leq sleq 1, 0leq tleq2pi) .$$
                In order to find the area of this floppy disc $F$ we have to compute
                $${bf r}_s=(cos t,sin t, 6scos tsin t),quad {bf r}_t=bigl(-ssin t,scos t ,3s^2cos(2t)bigr)$$
                and then
                $${bf r}_stimes{bf r}_t=(ldots,ldots,ldots) .$$
                The area is then finally given as
                $${rm area}(F)=int_0^1int_0^{2pi}bigl|{bf r}_stimes{bf r}_tbigr|>dt>ds .$$
                The resulting integral will be simpler than dreaded.






                share|cite|improve this answer














                Since in the end you have to integrate over the unit disc in the $(x,y)$-plane your source proposes to use polar coordinates instead of $x$ and $y$ as parameters. Then $x=scos t$, $y=ssin t$. In this way the idea $z=3xy$ $>(x^2+y^2leq1)$ translates into
                $${bf r}(s,t)=(scos t,ssin t,3s^2cos tsin t)qquad(0leq sleq 1, 0leq tleq2pi) .$$
                In order to find the area of this floppy disc $F$ we have to compute
                $${bf r}_s=(cos t,sin t, 6scos tsin t),quad {bf r}_t=bigl(-ssin t,scos t ,3s^2cos(2t)bigr)$$
                and then
                $${bf r}_stimes{bf r}_t=(ldots,ldots,ldots) .$$
                The area is then finally given as
                $${rm area}(F)=int_0^1int_0^{2pi}bigl|{bf r}_stimes{bf r}_tbigr|>dt>ds .$$
                The resulting integral will be simpler than dreaded.







                share|cite|improve this answer














                share|cite|improve this answer



                share|cite|improve this answer








                edited 15 hours ago

























                answered 18 hours ago









                Christian Blatter

                170k7111324




                170k7111324






























                     

                    draft saved


                    draft discarded



















































                     


                    draft saved


                    draft discarded














                    StackExchange.ready(
                    function () {
                    StackExchange.openid.initPostLogin('.new-post-login', 'https%3a%2f%2fmath.stackexchange.com%2fquestions%2f3004766%2fhow-to-calculate-the-surface-area-of-parametric-surface%23new-answer', 'question_page');
                    }
                    );

                    Post as a guest















                    Required, but never shown





















































                    Required, but never shown














                    Required, but never shown












                    Required, but never shown







                    Required, but never shown

































                    Required, but never shown














                    Required, but never shown












                    Required, but never shown







                    Required, but never shown







                    Popular posts from this blog

                    Can a sorcerer learn a 5th-level spell early by creating spell slots using the Font of Magic feature?

                    Does disintegrating a polymorphed enemy still kill it after the 2018 errata?

                    A Topological Invariant for $pi_3(U(n))$